Last visit was: 19 Nov 2025, 08:18 It is currently 19 Nov 2025, 08:18
Close
GMAT Club Daily Prep
Thank you for using the timer - this advanced tool can estimate your performance and suggest more practice questions. We have subscribed you to Daily Prep Questions via email.

Customized
for You

we will pick new questions that match your level based on your Timer History

Track
Your Progress

every week, we’ll send you an estimated GMAT score based on your performance

Practice
Pays

we will pick new questions that match your level based on your Timer History
Not interested in getting valuable practice questions and articles delivered to your email? No problem, unsubscribe here.
Close
Request Expert Reply
Confirm Cancel
605-655 Level|   Strengthen|                              
User avatar
ChiranjeevSingh
Joined: 22 Oct 2012
Last visit: 18 Nov 2025
Posts: 411
Own Kudos:
3,060
 [2]
Given Kudos: 154
Status:Private GMAT Tutor
Location: India
Concentration: Economics, Finance
Schools: IIMA  (A)
GMAT Focus 1: 735 Q90 V85 DI85
GMAT Focus 2: 735 Q90 V85 DI85
GMAT Focus 3: 735 Q88 V87 DI84
GMAT 1: 780 Q51 V47
GRE 1: Q170 V168
Expert
Expert reply
Schools: IIMA  (A)
GMAT Focus 3: 735 Q88 V87 DI84
GMAT 1: 780 Q51 V47
GRE 1: Q170 V168
Posts: 411
Kudos: 3,060
 [2]
2
Kudos
Add Kudos
Bookmarks
Bookmark this Post
User avatar
jabhatta2
Joined: 15 Dec 2016
Last visit: 21 Apr 2023
Posts: 1,294
Own Kudos:
Given Kudos: 188
Posts: 1,294
Kudos: 317
Kudos
Add Kudos
Bookmarks
Bookmark this Post
User avatar
KarishmaB
Joined: 16 Oct 2010
Last visit: 18 Nov 2025
Posts: 16,267
Own Kudos:
Given Kudos: 482
Location: Pune, India
Expert
Expert reply
Active GMAT Club Expert! Tag them with @ followed by their username for a faster response.
Posts: 16,267
Kudos: 76,994
Kudos
Add Kudos
Bookmarks
Bookmark this Post
User avatar
bv8562
Joined: 01 Dec 2020
Last visit: 01 Oct 2025
Posts: 423
Own Kudos:
Given Kudos: 360
GMAT 1: 680 Q48 V35
GMAT 1: 680 Q48 V35
Posts: 423
Kudos: 490
Kudos
Add Kudos
Bookmarks
Bookmark this Post
MartyTargetTestPrep GMATNinja What is the basis for eliminating option A?
User avatar
GMATNinja
User avatar
GMAT Club Verbal Expert
Joined: 13 Aug 2009
Last visit: 19 Nov 2025
Posts: 7,443
Own Kudos:
69,784
 [1]
Given Kudos: 2,060
Status: GMAT/GRE/LSAT tutors
Location: United States (CO)
GMAT 1: 780 Q51 V46
GMAT 2: 800 Q51 V51
GRE 1: Q170 V170
GRE 2: Q170 V170
Products:
Expert
Expert reply
GMAT 2: 800 Q51 V51
GRE 1: Q170 V170
GRE 2: Q170 V170
Posts: 7,443
Kudos: 69,784
 [1]
1
Kudos
Add Kudos
Bookmarks
Bookmark this Post
bv8562

MartyTargetTestPrep GMATNinja What is the basis for eliminating option A?­
The author gives us a reason why bacteria are more resistant to perxicillin: patients fail to take it as prescribed. We're trying to support that argument.

Here's (A):
Quote:

Resistance to several other commonly prescribed antibiotics has not increased since 1990 in Aqadestan
According to (A), the issue with perxicillin is kind of unique -- several other antibiotics aren't facing the same issue.

What (A) doesn't tell us is why there's a difference between perxicillin and these other drugs. Are patients taking the other drugs correctly, but taking perxicillin incorrectly? If so, that would strengthen the author's argument by pointing the finger at this issue. Alternatively, are patients taking all of the drugs (perxicillin and other) incorrectly? If so, that would weaken the author's argument, because something else must be causing the issue with perxicillin.

Because we have no idea why there's a difference between perxicillin and the other drugs, we can't say that (A) strengthens the author's argument.

Eliminate (A).

I hope that helps!­
avatar
Guest96
Joined: 11 Jan 2021
Last visit: 10 Oct 2025
Posts: 69
Own Kudos:
Given Kudos: 404
Location: India
Concentration: Strategy, Social Entrepreneurship
GPA: 4
WE:Analyst (Internet and New Media)
Posts: 69
Kudos: 19
Kudos
Add Kudos
Bookmarks
Bookmark this Post
WillGetIt
Since 1990 the percentage of bacterial sinus infections in Aqadestan that are resistant to the antibiotic perxicillin has increased substantially. Bacteria can quickly develop resistance to an antibiotic when it is prescribed indiscriminately or when patients fail to take it as prescribed. Since perxicillin has not been indiscriminately prescribed, health officials hypothesize that the increase in perxicillin resistant sinus infections is largely due to patients’ failure to take this medication as prescribed.

Which of the following, it true of Aqadestan, provides most support for the health officials' hypothesis:

(A) Resistance to several other commonly prescribed antibiotics has not increased since 1990 in Aqadestan

(D) Many patients who take perxicillin experience severe side effects within the first few days of their prescribed regimen.


CR06422

GMATNinja KarishmaB
I read both of your explanations but I still had some issues with understanding why (A) is wrong. Here are my POVs:

Option (A), as is, slightly strengthens the argument by conveying that antibiotic resistance is not a trend that's happening for all antibiotics. The negation of Option (A) slightly weakens the argument by conveying that there might be a trend (though what that trend is is not mentioned).
Doesn't the negated version then indirectly convey that something else (unmentioned) caused an increase in resistance? And hence becomes a strengthener/assumption.

Option (D) uses weak qualifiers "Many" and "within a few days of their regimen" to indirectly strengthen the argument by pointing out a reason for the patient's negligence.
I understand that correct options can be created with weak qualifiers, but when I compared the two options relatively, I felt that (A) was stronger.
User avatar
GMATNinja
User avatar
GMAT Club Verbal Expert
Joined: 13 Aug 2009
Last visit: 19 Nov 2025
Posts: 7,443
Own Kudos:
69,784
 [2]
Given Kudos: 2,060
Status: GMAT/GRE/LSAT tutors
Location: United States (CO)
GMAT 1: 780 Q51 V46
GMAT 2: 800 Q51 V51
GRE 1: Q170 V170
GRE 2: Q170 V170
Products:
Expert
Expert reply
GMAT 2: 800 Q51 V51
GRE 1: Q170 V170
GRE 2: Q170 V170
Posts: 7,443
Kudos: 69,784
 [2]
2
Kudos
Add Kudos
Bookmarks
Bookmark this Post
SlowTortoise
WillGetIt
Since 1990 the percentage of bacterial sinus infections in Aqadestan that are resistant to the antibiotic perxicillin has increased substantially. Bacteria can quickly develop resistance to an antibiotic when it is prescribed indiscriminately or when patients fail to take it as prescribed. Since perxicillin has not been indiscriminately prescribed, health officials hypothesize that the increase in perxicillin resistant sinus infections is largely due to patients’ failure to take this medication as prescribed.

Which of the following, it true of Aqadestan, provides most support for the health officials' hypothesis:

(A) Resistance to several other commonly prescribed antibiotics has not increased since 1990 in Aqadestan

(D) Many patients who take perxicillin experience severe side effects within the first few days of their prescribed regimen.

CR06422

GMATNinja KarishmaB

I read both of your explanations but I still had some issues with understanding why (A) is wrong. Here are my POVs:

Option (A), as is, slightly strengthens the argument by conveying that antibiotic resistance is not a trend that's happening for all antibiotics. The negation of Option (A) slightly weakens the argument by conveying that there might be a trend (though what that trend is is not mentioned).

Doesn't the negated version then indirectly convey that something else (unmentioned) caused an increase in resistance? And hence becomes a strengthener/assumption.

Option (D) uses weak qualifiers "Many" and "within a few days of their regimen" to indirectly strengthen the argument by pointing out a reason for the patient's negligence.

I understand that correct options can be created with weak qualifiers, but when I compared the two options relatively, I felt that (A) was stronger.
This question is creeping me out a little bit: at the moment, I happen to have a sinus infection that appears to be antibiotic-resistant. :shock: :shock: :shock:

Anyway, the hypothesis in the argument is that patient compliance is to blame for the issue with perxicillin. There's nothing implying that this is ONLY an issue with perxicillin -- perhaps patients stink at following directions with ALL antibiotics. Or, maybe it IS just perxicillin, and patients are great at taking other antibiotics for some reason. Either way, it wouldn't impact the hypothesis that the patients are to blame. So, the presence or absence of a "trend" wouldn't support the argument.

(A) tells us that resistance to several other commonly prescribed antibiotics has not increased since 1990 in Aqadestan. But again, maybe patients are better at following directions with these other antibiotics, but are not good at sticking to the plan with perxicillin. (A) doesn't give us any insight into whether patient compliance is the problem with perxicillin, so we can eliminate it.

Incidentally, (D) provides us with a reason why patients specifically are to blame: many patients experience severe side effects, which explains why they don't take the drug as prescribed.

That's why (D) is the correct answer.

I hope that helps!
User avatar
jain67
Joined: 17 Jan 2022
Last visit: 09 Nov 2024
Posts: 52
Own Kudos:
Given Kudos: 52
Location: India
Concentration: Marketing, Leadership
GMAT Focus 1: 645 Q85 V81 DI80
Products:
GMAT Focus 1: 645 Q85 V81 DI80
Posts: 52
Kudos: 33
Kudos
Add Kudos
Bookmarks
Bookmark this Post
MartyMurray GMATNinja Option B can have 2 directions- 1. People do not consult doctors and hence they are not prescribed antibiotics. In this case, we can eliminate B.
2. People don't consult doctors and consume antibiotics without a prescription. This becomes our strengthner.
So how do we go about it?
User avatar
GMATNinja
User avatar
GMAT Club Verbal Expert
Joined: 13 Aug 2009
Last visit: 19 Nov 2025
Posts: 7,443
Own Kudos:
Given Kudos: 2,060
Status: GMAT/GRE/LSAT tutors
Location: United States (CO)
GMAT 1: 780 Q51 V46
GMAT 2: 800 Q51 V51
GRE 1: Q170 V170
GRE 2: Q170 V170
Products:
Expert
Expert reply
GMAT 2: 800 Q51 V51
GRE 1: Q170 V170
GRE 2: Q170 V170
Posts: 7,443
Kudos: 69,784
Kudos
Add Kudos
Bookmarks
Bookmark this Post
jain67
MartyMurray GMATNinja Option B can have 2 directions- 1. People do not consult doctors and hence they are not prescribed antibiotics. In this case, we can eliminate B.

People don't consult doctors and consume antibiotics without a prescription. This becomes our strengthner.

So how do we go about it?

­The hypothesis is that the increase in perxicillin-resistant sinus infections is largely due to patients’ failure to take the medication as prescribed.

So if Tim decides to take the medication WITHOUT a prescription, then there's no way to say that he has or has not taken it "as prescribed" because it wasn't prescribed at all!

The prescription represents the instructions from the medical professional, and we can't say whether or not a patient has followed the instructions if the instructions never existed.

Your second scenario might support a DIFFERENT hypothesis (i.e. that the increase in perxicillin-resistant sinus infections is largely due to patients' taking the medication WITHOUT a prescription), but it does not support the specific hypothesis in the passage.

(D) is a much better answer, as explained here: https://gmatclub.com/forum/since-1990-the-percentage-of-bacterial-sinus-infections-in-aqadestan-199805-20.html#p3328065
User avatar
Kavicogsci
Joined: 13 Jul 2024
Last visit: 09 Feb 2025
Posts: 167
Own Kudos:
Given Kudos: 154
GMAT 1: 710 Q48 V40
GMAT 1: 710 Q48 V40
Posts: 167
Kudos: 91
Kudos
Add Kudos
Bookmarks
Bookmark this Post
MartyMurray GMATNinja AnishPassi
Ik tons of explanations but why can't A still be a defender strengthener - by removing an additional cause that there hasn't been an overall decline in the trend of resistance we can single out and blame patients prescription compliance for this one. Ofc one may doubt and say hey but what if they is no resistance for other antibiotics because patients are following prescriptions in first place for all of them and therefore we have no reason to doubt that they may not follow here, but i can make a similar argument for option D that despite severe complications, i still dont stop the prescription
User avatar
GMATNinja
User avatar
GMAT Club Verbal Expert
Joined: 13 Aug 2009
Last visit: 19 Nov 2025
Posts: 7,443
Own Kudos:
69,784
 [1]
Given Kudos: 2,060
Status: GMAT/GRE/LSAT tutors
Location: United States (CO)
GMAT 1: 780 Q51 V46
GMAT 2: 800 Q51 V51
GRE 1: Q170 V170
GRE 2: Q170 V170
Products:
Expert
Expert reply
GMAT 2: 800 Q51 V51
GRE 1: Q170 V170
GRE 2: Q170 V170
Posts: 7,443
Kudos: 69,784
 [1]
1
Kudos
Add Kudos
Bookmarks
Bookmark this Post
Kavicogsci

Ik tons of explanations but why can't A still be a defender strengthener - by removing an additional cause that there hasn't been an overall decline in the trend of resistance we can single out and blame patients prescription compliance for this one. Ofc one may doubt and say hey but what if they is no resistance for other antibiotics because patients are following prescriptions in first place for all of them and therefore we have no reason to doubt that they may not follow here, but i can make a similar argument for option D that despite severe complications, i still dont stop the prescription
You have to ask yourself: "Without any other information, how does this impact the argument?"

With choice (D), knowing that there are SEVERE side effects is reason to believe that people will stop taking the medicine. Sure, maybe those people will continue taking the medication. But by itself, choice (D) is EVIDENCE that people will stop taking the medication. (For more on that, check out this post.)

The problem with (A) is that it could go either way. Imagine that resistance to the other commonly prescribed antibiotics HAS gone up. We still don't know WHY resistance to any of the individual antibiotics went up.

Maybe resistance to some went up because they were prescribed indiscriminately, and maybe resistance to others went up because patients failed to take them as prescribed. Do we put perxicillin in the first group or the second? There's no evidence one way or the other, so we have no idea.

In short, the resistance to other antibiotics doesn't tell us WHY people are resistant to perxicillin, so (A) is irrelevant.
User avatar
Mantrix
Joined: 13 May 2023
Last visit: 19 Nov 2025
Posts: 163
Own Kudos:
Given Kudos: 34
Location: India
GMAT Focus 1: 595 Q87 V75 DI77
GMAT Focus 2: 625 Q81 V82 DI80
GPA: 9
GMAT Focus 2: 625 Q81 V82 DI80
Posts: 163
Kudos: 122
Kudos
Add Kudos
Bookmarks
Bookmark this Post
P1: Since 1990 the percentage of bacterial sinus infections in A that are resistant to the antibiotic p has increased substantially.
P2: Bacteria can quickly develop resistance to an antibiotic when
1. it is prescribed indiscriminately or
2. when patients fail to take it as prescribed.

P3: Since p has not been indiscriminately prescribed,
C: the increase in p resistant sinus infections is largely due to patients’ failure to take this medication as prescribed.

Strengthen the conclusion:

(A) Resistance to several other commonly prescribed antibiotics has not increased since 1990 in A
we are not concerned about other[b]Rejected

(B) A large number of A never seek medical help when they have a sinus infection. (If they don't seek the help, means medication is not prescribed to the patients), so there must be other reason for that

(C) When it first became available, p was much more effective in treating bacterial sinus infections than any other antibiotic used for such infections at the time.(not relevant)

(D) Many patients who take p experience severe side effects within the first few days of their prescribed regimen
(so, if they feel side effects they will stop it, so yeah, they didn't take it as prescribed, as they must take some break in between), Strengthen

(E) Health clinics provide antibiotics to their patients at cost
(not relevant)[/b]
User avatar
Aishna1034
Joined: 21 Feb 2023
Last visit: 19 Nov 2025
Posts: 219
Own Kudos:
Given Kudos: 150
Products:
Posts: 219
Kudos: 64
Kudos
Add Kudos
Bookmarks
Bookmark this Post
KarishmaB Could you please elaborate on option B. If they dont take medical practitioner's help, then there are chances they are not taking it as prescribed( if they would have consulted the practitioner). Isnt this giving a strengthening possibility?
KarishmaB


"Patients do not take it as prescribed" is not an assumption. It is the given conclusion. We need to strengthen this conclusion so we need to increase the probability that it is correct. Now, if we are given a reason why people may not take it as prescribed, it will strengthen our conclusion.
Severe side effects could be one such reason. It increases the probability that people do not take it as prescribed.
Another reason could be say one needs to take it every 3 hrs so people skip it at night. Again, gives a reason why people may not take it as prescribed and could be another strengthener.
User avatar
egmat
User avatar
e-GMAT Representative
Joined: 02 Nov 2011
Last visit: 19 Nov 2025
Posts: 5,108
Own Kudos:
Given Kudos: 700
GMAT Date: 08-19-2020
Expert
Expert reply
Active GMAT Club Expert! Tag them with @ followed by their username for a faster response.
Posts: 5,108
Kudos: 32,886
Kudos
Add Kudos
Bookmarks
Bookmark this Post
Aishna1034
KarishmaB Could you please elaborate on option B. If they dont take medical practitioner's help, then there are chances they are not taking it as prescribed( if they would have consulted the practitioner). Isnt this giving a strengthening possibility?

Aishna1034 I'll add my two cents here. You're missing a critical distinction.

Option B talks about people who never seek medical help for sinus infections. This means they:
- Never get prescribed perxicillin
- Never take perxicillin at all
- Cannot contribute to perxicillin resistance

Why Your Logic Breaks Down:

You're thinking: "If they don't consult a doctor → they might self-medicate incorrectly → this strengthens the hypothesis"

But here's the problem: Bacteria can only develop resistance to an antibiotic when they're actually exposed to it. People who never seek medical help aren't taking perxicillin (prescribed or otherwise), so their bacteria never encounter the drug and can't develop resistance to it.

The Hypothesis's Specific Focus:

The health officials' hypothesis is about prescribed perxicillin users who don't complete their course properly. It's this group that creates resistant bacteria by:
  1. Starting the antibiotic (killing weaker bacteria)
  2. Stopping early (allowing stronger bacteria to survive and multiply)
  3. Creating a resistant strain

Option B describes a completely different population that's irrelevant to the resistance problem.

Strategic Pattern Recognition:

In GMAT CR strengthen/weaken questions about cause-and-effect:
- Always identify the specific population the conclusion addresses
- Eliminate options about different populations (like B)
- Focus on options that directly impact the causal mechanism (like D, which explains why patients stop taking medication early)

You can practice similar Critical Reasoning questions here (these are official questions) - select Strengthen/Weaken under CR and choose the difficulty level based on your current understanding.
   1   2 
Moderators:
GMAT Club Verbal Expert
7443 posts
GMAT Club Verbal Expert
231 posts
189 posts